Pls help me it’s due tmrw

Pls Help Me Its Due Tmrw

Answers

Answer 1

Answer:

Problem 8:
Option C)       DE = √21

Problem 9:
Option C)     Perimeter = 30 cm

Step-by-step explanation:

1. Problem 8

Draw a line segment connecting the center of the circle, X, to A. This is the radius of the circle

The points ABX form a right triangle with AX as the hypotenuse and AB, BX as the legs of the right triangle

By the Pythagorean theorem
hypotenuse² = sum of the squares of the two legs

Plugging in line segment references

AX² = AB² + BX²

We are given AC = 8, BX = 3

Since the segment BX intersects AC at right angles, AB = BC = AC/2

So AB = 8/2= 4

Plugging these values into the Pythagorean formula
AX² = AB² + BX²

AX² = 4² +3²

AX² = 16 + 9

AX² = 25

Draw a line connecting X and D
The points DEX form a right triangle with DX as the hypotenuse and EX and DE as the legs

Again, by the Pythagorean theorem

DX² = DE² + EX²

But DX is the radius of the circle so it must be the same as AX

Substitute for DX in terms of AX
DX² = DE² + EX²

=> AX² = DE² + EX²

But AX² = 25 and EX = 2 giving EX² = 4

Therefore

AX² = DE² + EX² becomes
25 = DE² + 4

DE² = 25 - 4

DE² = 21

DE = √21

This is choice C

Problem 9

To find the perimeter, just add up the individual side lengths:

Perimeter = 10 + 8 + 7 + 5 = 30 cm

Option C


Related Questions

Lines ab and cd are parallel. if 6 measures (4x - 31)°, and 5 measures 95°, what is the value of x? a. x = 19 b. x = 95 c. x = 265 d. x = 29

Answers

Answer: x=29

Step-by-step explanation:

To find the value of x, we can set the two angles equal to each other and solve for x, which gives x = 19.

What will be the value of x if 6 measures (4x - 31)° and 5 measures 95° in parallel lines ab and cd?

We can use the fact that alternate interior angles are congruent when a transversal intersects parallel lines. In this case, line ab and cd are parallel and 6 and 5 are alternate interior angles. So we can set up an equation:

4x - 31 = 95

Solving for x:

4x = 126

x = 31.5

So the value of x is not one of the answer choices given. However, if we round x to the nearest integer, we get x = 32, which is closest to answer choice (d) x = 29. Therefore, the closest answer choice is (d) x = 29.

Learn more about interior angles

brainly.com/question/10638383

#SPJ11

The function R = 73. 3*/M3, known as Kielber's law, relates the basal metabolic rate R In Calories per day


burned and the body mass M of a mammal In kilograms.


a. Find the basal metabolic rate for a 180 kilogram lion. Then find the formula's prediction for a 80


kilogram human. If necessary round down to the nearest 50 Calories.


b. Use your metabolic rate result for the lion to find what the basal metabolic rate for a 80 kllogram


human would be if metabolic rate and mass were directly proportional. Compare the result to the result


from part a.


a. Kleiber's law for lion


Calories


Kleiber's law for humans


Calories


b. If metabolic rate and mass were directly proportional


Calories


If the metabolic rate were directly proportional to mass, then the rate for a human would be


(select)


than the actual prediction from Kleiber's law. Kleiber's law Indicates that smaller


organisms have a (select) v metabolic rate per kilogram of mass than do larger organisms.

Answers

The basal metabolic rate for a 180-kilogram lion is approximately 766.4 Calories per day.

The formula predicts that an 80-kilogram human would have a basal metabolic rate of approximately 1,313.9 Calories per day.

The basal metabolic rate is the amount of energy that an organism needs to carry out its basic physiological functions, such as breathing and circulating blood. In this case, Kielber's law is expressed as:

R = 73 [tex]\sqrt[4]{M^3}[/tex]

Let's use this function to find the basal metabolic rate for a 180-kilogram lion. To do this, we simply substitute M = 180 into the equation and solve for R:

R = 73 [tex]\sqrt[4]{180^3}[/tex]

R = 73 [tex]\sqrt[4]{5832}[/tex]

R ≈ 766.4

Now, let's find the formula's prediction for an 80-kilogram human. Again, we simply substitute M = 80 into the equation and solve for R:

R = 73[tex]\sqrt[4]{80^3}[/tex]

R = 73[tex]\sqrt[4]{512}[/tex]

R ≈ 1,313.9

To know more about function here

https://brainly.com/question/28193995

#SPJ4

Complete Question:

The function R = 73 [tex]\sqrt[4]{M^3}[/tex], known as Kielber's law, relates the basal metabolic rate R In Calories per day burned and the body mass M of a mammal In kilograms.

Find the basal metabolic rate for a 180-kilogram lion. Then find the formula's prediction for an 80-kilogram human. If necessary round down to the nearest 50 Calories.

Can someone help me asap? It’s due today!! Show work! I will give brainliest if it’s correct and has work

Make a probability table!

Answers

The probability of choosing randomly with replacement an H or P in either selection is derived to be equal to 0.16 which makes the last option correct.

What is probability

The probability of an event occurring is the fraction of the number of required outcome divided by the total number of possible outcomes.

The total possible outcome = 5

the event of selecting H = 1

probability of selecting H= 1/5

the event of selecting P = 2

probability of selecting H= 2/5

probability of choosing an H or P in either selection = 1/5 × 2/5 + 2/5 × 1/5

probability of choosing an H or P in either selection = 4/25

probability of choosing an H or P in either selection = 0.16

Therefore, the probability of choosing randomly with replacement an H or P in either selection is derived to be equal to 0.16

Read more about probability here:https://brainly.com/question/251701

#SPJ1

In an isosceles triangle, the measure of a base angle is 65. Find the number of degrees in the measure of the vertex angle

Answers

The number of degrees in the measure of the vertex angle is 50 degrees.

An isosceles triangle has two equal sides and two equal base angles. In your question, the measure of a base angle is 65 degrees. To find the measure of the vertex angle, we'll use the fact that the sum of angles in any triangle is always 180 degrees.

Since both base angles are equal, their combined measure is 2 * 65 = 130 degrees. Now, we subtract the sum of the base angles from the total angle measure of the triangle:

180 degrees (total angle measure) - 130 degrees (sum of base angles) = 50 degrees.

So, the measure of the vertex angle in the isosceles triangle is 50 degrees. In summary, when given the measure of a base angle in an isosceles triangle, we can use the triangle's angle sum property to find the measure of the vertex angle.

Learn more about isosceles triangle here: https://brainly.com/question/29793403

#SPJ11

What expression represents the volume of the cylinder, in cubic units? 4πx2 2πx3 πx2 2x 2 πx3

Answers

The expression that represents the volume of the cylinder, in cubic units, is:

[tex]$$V = 2\pi x^3$$[/tex]

The expression that represents the volume of a cylinder in cubic units is given by the formula:

[tex]$$V = \pi r^2h$$[/tex]

where [tex]$r$[/tex] is the radius of the base of the cylinder and [tex]$h$[/tex] is the height of the cylinder.

Now, let's consider each option provided:

[tex]1. $4\pi x^2$[/tex]

This expression only includes the radius, but it does not include the height of the cylinder, so it cannot be the correct answer.

[tex]2. $2\pi x^3$[/tex]

This expression includes both the radius and the height of the cylinder, but it does not include the squared term for the radius, so it cannot be the correct answer.

[tex]3. $\pi x^2$[/tex]

This expression includes the squared term for the radius, but it does not include the height of the cylinder, so it cannot be the correct answer.

[tex]4. $2x$[/tex]

This expression only includes a single variable, which is neither the radius nor the height of the cylinder, so it cannot be the correct answer.

[tex]5. $2\pi x^3$[/tex]

This expression includes both the squared term for the radius and the height of the cylinder, so it is the correct answer.

Therefore, the expression that represents the volume of the cylinder, in cubic units, is:

[tex]$$V = 2\pi x^3$$[/tex]

This formula can be used to calculate the volume of a cylinder given the value of its radius and height.

To learn more about  volume of the cylinder refer here:

https://brainly.com/question/29762858

#SPJ11

x Which statement about prime and composite numbers is true?
x
A The product of any two prime numbers is a prime number.
* B The product of any two prime numbers is a composite number.
* C All prime numbers are odd numbers.
√x
D All even numbers are composite numbers.

Answers

B, as 2 primes mean another solution to get a composite number. Ex: 7 and 3. Together they make 21, and 21 can be divided into 2 things: 1 and 21 along with 7 and 3

consider the following 8 numbers, where one labelled x is unknown. 26 , 7 , 17 , x , 21 , 6 , 34 , 27 given that the range of the numbers is 63, work out 2 values of x .

Answers

The two possible values of x are -29 and 69.

To find two possible values for x, we need to use the fact that the range of

the numbers is 63.

The range is defined as the difference between the largest and smallest

numbers in the set.

First, we can find the largest and smallest numbers in the set:

Smallest number = 6

Largest number = 34

Next, we can set up two equations to represent the range of the numbers,

using the two possible scenarios for x:

Scenario 1:

If x is the smallest number in the set, then the range is equal to [tex]34 - x.[/tex]

Scenario 2: If x is the largest number in the set, then the range is equal to

[tex]x - 6[/tex].

We can then set up two equations and solve for x in each scenario:

Scenario 1:

[tex]34 - x = 63x = 34 - 63x = -29[/tex]

Scenario 2:

[tex]x - 6 = 63x = 63 + 6x = 69[/tex]

To know more about range refer here https://brainly.com/question/28135761# #SPJ11

How do I take a picture

Answers

To take a picture we must press the shutter button pointing the lens towards the image we want to capture.

How to take a picture?

To take a picture we must follow the following steps. In general, we must have a camera at hand and know how to use it. There is a great diversity of cameras with different characteristics, but the basics to take a photo are the following:

In the first place, we must locate ourselves at a prudent distance from the element that we are going to photograph, making sure that it comes out completely in the camera's focus.

Once we have focused on the object, we must make sure that nothing is going to move the camera or go through between the camera and the object.

Later, we must make sure that there is enough light for the object to come out sharp in the photo.

Finally, we press the shutter and take the photo. In some cases we will have the digital photo or in others we will be able to print it on photographic paper.

Learn more about photography at: https://brainly.com/question/30685203

#SPJ1

HELP! WILL GIVE BRAINLEST! An angle of 1. 5 rad intercepts an arc on the unit circle. What is the length of the intercepted arc?

Answers

The length of the intercepted arc on the unit circle is equal to the radius of the circle times the angle in radians. In this case, since the unit circle has a radius of 1, the length of the intercepted arc is simply equal to the angle in radians.

So, the length of the intercepted arc for an angle of 1.5 radians is 1.5 units (since the angle is given in radians, not degrees).

To Know more about   length of the intercepted arc refer here

https://brainly.com/question/27602835#

#SPJ11

50 PONTS Triangle LMN has vertices at L(−1, 4), M(−1, 0), and N(−3, 4) Determine the vertices of image L′M′N′ if the preimage is rotated 90° clockwise about the origin.

L′(4, 1), M′(0, 1), N′(4, 3)
L′(−1, −4), M′(−1, 0), N′(−3, −4)
L′(−4, −1), M′(0, −1), N′(−4, −3)
L′(1, −4), M′(1, 0), N′(3, −4)

Answers

The coordinates of the resulting triangle are L'(4, 1), M'(0, 1), and N'(4, 3)

What are the coordinates of the resulting triangle?

From the question, we have the following parameters that can be used in our computation:

Triangle LMN has vertices at L(−1, 4), M(−1, 0), and N(−3, 4

This means that

L(−1, 4), M(−1, 0), and N(−3, 4Rotation rule = 90° clockwise around the origin.

The rotation rule of 90° clockwise around the origin is

(x,y) becomes (y,-x)

So, we have

Image = (y, -x)

Substitute the known values in the above equation, so, we have the following representation

L'(4, 1), M'(0, 1), and N'(4, 3)

Hence, the coordinates of the resulting points, are L'(4, 1), M'(0, 1), and N'(4, 3)

Read more about transformation at

brainly.com/question/27224272

#SPJ1

Answer:L′(4, 1), M′(0, 1), N′(4, 3)

Step-by-step explanation:

I am in the middle of taking the quiz and I believe this is the correct answer!

Lisa has 9 rings in her jewelry box. Five are gold and 4 are silver. If she randomly selects 3 rings to wear to a party, find each probability. P(2 silver or 2 gold)

Answers

The probability of selecting 2 silver rings or 2 gold rings is 3/28.

How to find the probability of selecting 2 silver rings or 2 gold rings?

To find the probability of selecting 2 silver rings or 2 gold rings, we need to find the probability of each event separately and then add them.

Probability of selecting 2 silver rings:

There are 4 silver rings out of 9 total, so the probability of selecting a silver ring on the first draw is 4/9. After the first ring is selected, there are 3 silver rings left out of 8 total, so the probability of selecting a second silver ring is 3/8. Finally, after two silver rings have been selected, there are 2 silver rings left out of 7 total, so the probability of selecting a third silver ring is 2/7. Therefore, the probability of selecting 2 silver rings is:

(4/9) * (3/8) * (2/7) = 24/504 = 1/21

Probability of selecting 2 gold rings:

Similarly, there are 5 gold rings out of 9 total, so the probability of selecting a gold ring on the first draw is 5/9. After the first ring is selected, there are 4 gold rings left out of 8 total, so the probability of selecting a second gold ring is 4/8 = 1/2. Finally, after two gold rings have been selected, there are 3 gold rings left out of 7 total, so the probability of selecting a third gold ring is 3/7. Therefore, the probability of selecting 2 gold rings is:

(5/9) * (1/2) * (3/7) = 15/126 = 5/42

Adding the probabilities of selecting 2 silver rings or 2 gold rings, we get:

P(2 silver or 2 gold) = P(2 silver) + P(2 gold) = 1/21 + 5/42 = 3/28

Therefore, the probability of selecting 2 silver rings or 2 gold rings is 3/28.

Learn more about probability

brainly.com/question/30034780

#SPJ11

Let AX = B be a consistent linear system with 12 equations and 8 variables. If the solution of the system contains 3 free variables, then what is the rank of the coefficient matrix A?

Answers

The rank of the coefficient matrix A is 5.

How to determined the matrix?

Since the system AX = B is consistent and has 12 equations and 8 variables, the rank of the coefficient matrix A must be less than or equal to 8 (the number of variables).

If the solution of the system contains 3 free variables, it means that the dimension of the null-space of A is 3. By the rank-nullity theorem,

we know that the dimension of the null-space of A plus the rank of A is equal to the number of columns of A (which is 8 in this case).

Therefore, we have:

rank(A) + dim(null(A)) = 8

rank(A) + 3 = 8

rank(A) = 5

So, the rank of the coefficient matrix A is 5.

Learn more about matrix

brainly.com/question/9967572

#SPJ11

The height, h, in feet of a ball suspended from a spring as a function of time, t, in seconds can be modeled by the equation h = negative 2 sine (pi (t one-half)) 5. which of the following equations can also model this situation? h = negative 2 cosine (pi t) 5 h = negative 2 cosine (pi (t one-half)) 5 h = 2 cosine (pi t) 5 h = 2 cosine (pi (t one-half)) 5

Answers

The correct answer for the equation is  [tex]h = -2cos(\pi t) + 5[/tex] . The correct option is (1)

Given:

[tex]h= -2sin(\pi\tfrac{t}{2} )[/tex]

Examine the answer choices:

[tex]h = -2cos(\pi t) + 5[/tex]

Amplitude: |-2| = 2 (same as the given equation)

Frequency: π (same as the given equation)

Phase Shift: None (different from the given equation)

[tex]h = -2cos(\pi (t/2)) + 5[/tex]

Amplitude: |-2| = 2 (same as the given equation)

Frequency: π/2 (different from the given equation)

Phase Shift: None (different from the given equation)

[tex]h = 2cos(\pi t) + 5[/tex]

Amplitude: |2| = 2 (different from the given equation)

Frequency: π (same as the given equation)

Phase Shift: None (different from the given equation)

[tex]h = 2cos(\pi(t/2)) + 5[/tex]

Amplitude: |2| = 2 (different from the given equation)

Frequency: π/2 (different from the given equation)

Phase Shift: None (different from the given equation)

The correct equation is [tex]h = -2cos(\pi t) + 5[/tex]  .The correct option is (1).

Learn more about Equation here:

https://brainly.com/question/27767092

#SPJ12

Penelope invested $89,000 in an account paying an interest rate of 6}% compounded continuously. Samir invested $89,000 in an account paying an interest rate of 6⅜% compounded monthly. To the nearest hundredth of a year, how much longer would it take for Samir's money to double than for Penelupe's money to double?

Answers

To solve the problem, we need to find out how much longer it would take for Samir's money to double compared to Penelope's money, given that Penelope invested $89,000 in an account with a continuous interest rate of 6%, while Samir invested $89,000 in an account with a monthly compounded interest rate of 6⅜%.

For Penelope's investment, we can use the formula for continuous compounding, which is A = Pe^(rt), where A is the amount of money after t years, P is the initial investment, r is the interest rate as a decimal, and e is the natural logarithm base. We know that Penelope invested $89,000 and we want to find t such that A = 2P = $178,000. Thus, we have:

$178,000 = $89,000e^(0.06t)

Dividing both sides by $89,000 and taking the natural logarithm of both sides, we get:

ln(2) = 0.06t

Solving for t, we get:

t = ln(2)/0.06 ≈ 11.55 years

For Samir's investment, we can use the formula for monthly compounded interest, which is A = P(1 + r/12)^(12t), where A, P, r are the same as before, and t is the time in years divided by 12. Similarly, we know that Samir invested $89,000 and we want to find t such that A = 2P = $178,000. Thus, we have:

$178,000 = $89,000(1 + 0.0638/12)^(12t)

Dividing both sides by $89,000 and taking the logarithm (base 1 + r/12) of both sides, we get:

log(2)/log(1 + 0.0638/12) = 12t

Solving for t, we get:

t ≈ 11.80/12 = 0.98 years

To find the difference in time it takes for Samir's money to double compared to Penelope's, we subtract the time it takes for Penelope's money to double from the time it takes for Samir's money to double:

0.98 - 11.55 ≈ -10.57

However, this answer doesn't make sense in the context of the problem, since it's negative. After reviewing our solution, we realized that we made a mistake in the calculation of t for Penelope's investment. We need to find the time it takes for Penelope's investment to double with annual compounding, not continuous compounding. The formula for this is t = (ln(2))/(ln(1 + r)), where r is the annual interest rate as a decimal.

Plugging in the numbers, we get:

t = (ln(2))/(ln(1 + 0.06)) ≈ 11.55 years

This is the same as the time we got for Samir's investment, so the difference in time it takes for their money to double is:

0.98 - 11.55 ≈ -10.57

Again, this answer doesn't make sense in the context of the problem, since it's negative. Therefore, we need to revise our solution and approach the problem differently.

1. at which location in new york state


would one least expect to find fossils in


the surface bedrock?

Answers

One would least expect to find fossils in the surface bedrock in the Adirondack Mountains region of New York State.

This region is known for having some of the oldest rocks in North America, dating back over a billion years. These rocks were formed through volcanic activity and mountain-building processes that occurred long before the evolution of complex life forms.

As a result, the rocks in the Adirondack Mountains are generally not rich in fossils, especially those of plants and animals that evolved much later in Earth's history.

In contrast, other regions of New York State, such as the Hudson Valley and the Finger Lakes region, have rocks that are more conducive to fossil preservation. These regions were covered by shallow seas at various times in the past, allowing for the accumulation of sediment and the preservation of fossils of marine organisms.

To learn more about fossil, click here:

https://brainly.com/question/31419516

#SPJ11

Can u please help me solve this and explain how you got it please.

8xsquared-2-5x=8

Find the x

Answers

Using quadratic formula, the value of x in the quadratic equation are 1.47 and -0.85

What is the value of x?

To find the value of x, we can either use quadratic formula or factorization method.

8x² - 2 - 5x = 8

Let's rewrite the equation properly

8x² - 5x - 2 - 8 = 0

8x² - 5x - 10 = 0

a = 8, b = -5, c = -10

Using quadratic formula;

-b ±[√b² - 4ac / 2a]

-(-5) ±[√(-5)² - 4(8)(-10) / 2(8)]

x = 5+ √345 / 16 or x = 5 - √345 / 16

x = 1.47 or x = -0.85

Learn more on quadratic equations here;

https://brainly.com/question/1214333

#SPJ1

Which answer gives the correct transformation of P(x) to get to I(x)?



A. ) I(x)=P(1/2x)


B. ) I(x)=P(2x)


C. ) I(x)=1/2P(x)


D. ) I(x)=2P(x)

Answers

The answer that gives the correct transformation of P(x) to get to I(x) is option D) I(x) = 2P(x).

This means that the function I(x) is obtained by multiplying the function P(x) by 2.

To understand why this is the correct transformation, let's consider an example:

Suppose P(x) represents the number of items produced by a factory in x hours. If we want to find the number of items produced by the factory in 2x hours, we can use the transformation I(x) = 2P(x). This is because the rate of production is constant, so in twice the time, the factory will produce twice the number of items. Therefore, multiplying the function P(x) by 2 gives us the function I(x) that represents the number of items produced by the factory in 2x hours.

Option A) I(x) = P(1/2x) means that we are compressing the function P(x) horizontally, which would result in a faster rate of change. This transformation does not make sense in the context of the problem and is not the correct transformation.

Option B) I(x) = P(2x) means that we are stretching the function P(x) horizontally, which would result in a slower rate of change. This transformation also does not make sense in the context of the problem and is not the correct transformation.

Option C) I(x) = 1/2P(x) means that we are reducing the function P(x) by half, which would result in a slower rate of change. This transformation does not match the problem statement and is not the correct transformation.

To learn more about  transformation visit:

https://brainly.com/question/29641135

#SPJ11

At the toy store, 4 toy cars cost $3.24. How much does it cost to buy 25 toy cars?=

Answers

Answer:

Each car cost $0.81, you would need to do 0.81 times 25 and you would get $20.25

Step-by-step explanation:

The tangent plane to the surface with equation - in (9) +-3 at the point (z,y,z) - (2,1,9) has the equation ________.

Answers

The equation of the tangent plane to the surface with equation - in (9) +-3 at the point (2,1,9),  first need to find the partial derivatives of the function with respect to x, y, and z. However, the surface equation provided seems to be incorrect or incomplete. Please provide the correct surface equation in the form f(x, y, z) = constant.

Let's call the function f(x, y, z) = - in (9) +-3.
∂f/∂x = 0  (since there is no x term in the function)
∂f/∂y = 0  (since there is no y term in the function)
∂f/∂z = -3/((z-9)^2)
Now we can use the formula for the equation of the tangent plane at a point (a,b,c) on a surface z=f(x,y):
z - f(a,b) = (∂f/∂x)(a,b)(x-a) + (∂f/∂y)(a,b)(y-b)
+ (∂f/∂z)(a,b)(z-c)

Plugging in the values we have, we get:
z - (- in (9) +-3)|_(2,1) = 0(x-2) + 0(y-1) - (3/((z-9)^2))|_(2,1,9)(z-9)
Simplifying:
z + in (9) - 3 = -3(z-9)
4z = 30
z = 7.5
So the equation of the tangent plane is:
z - (- in (9) +-3)|_(2,1) = (-3/((z-9)^2))|_(2,1,9)(z-9)
z - in (9) - 3 = -3(7.5-9)
z - in (9) - 3 = 4.5
z = 12.5
Therefore, the equation of the tangent plane to the surface with equation - in (9) +-3 at the point (2,1,9) is:

z - in (9) - 3 = -3/((z-9)^2)(z-9)
z - in (9) - 3 = -3(z-9)/(z-9)^2
z - in (9) - 3 = -3/(z-9)
z = 3/(z-9) + in (9) + 3
or
3x + 3y - 4z = -27 + in (9)
To find the equation of the tangent plane to the surface with equation ln(9) +- 3 at the point (x, y, z) = (2, 1, 9), follow these steps:
1. Determine the gradient vector of the given surface at the point (2, 1, 9).
2. Use the gradient vector as the normal vector of the tangent plane.
3. Write the equation of the tangent plane using the normal vector and the given point.
However, the surface equation provided seems to be incorrect or incomplete. Please provide the correct surface equation in the form f(x, y, z) = constant, so that I can help you find the equation of the tangent plane.

Visit here to learn more about tangent plane:

brainly.com/question/30260323

#SPJ11

Which pair of lines in this figure are perpendicular?

A.
lines B and F

B.
lines F and D

C.
lines C and E

D.
lines A and D

Answers

Answer:

D. Lines A and D are perpendicular.

DAnswer:

Step-by-step explanation:

Sarah has a solid wooden cube with a length of 4/5 cm. From each of its 8 corners, she cuts out a smaller cube with a length of 1/5 cm. What is the volume of the block after cutting out the smaller cubes?

Answers

The volume of the block after cutting out the smaller cubes is 56/125 cubic centimeters.

The initial volume of the solid wooden cube is given by:

V_initial = (4/5 cm)³ = 64/125 cm³

To find the volume of each of the 8 smaller cubes cut out from the corners, we can use the formula:

V_small cube = (1/5 cm)³= 1/125 cm³

Since we cut out 8 smaller cubes, the total volume of the smaller cubes is:

V_small cubes = 8 x (1/125 cm³) = 8/125 cm³

To find the final volume of the block after cutting out the smaller cubes, we can subtract the volume of the smaller cubes from the initial volume of the block:

V_final = V_initial - V_small cubes

Substituting the values we obtained earlier, we get:

V_final = (64/125 cm³) - (8/125 cm³) = 56/125 cm³

Know more about volume here:

https://brainly.com/question/1578538

#SPJ11

What is the multiplicity of the zero of the polynomial function that represents the volume of a sphere with radius x+5

Answers

The graph of the function will touch the x-axis at x = -5, but not cross it, and the behavior of the graph near x = -5 will be determined by the degree of the zero (which is 3 in this case).

The polynomial function that represents the volume of a sphere with radius x+5 is given by:

[tex]V(x) = (4/3)\pi (x+5)^3[/tex]

To find the multiplicity of the zero, we need to factor out the (x+5) term from the polynomial:

V(x) = (4/3)π(x+5)(x+5)(x+5)

We can see that the zero is x = -5, and it has a multiplicity of 3, since there are three factors of (x+5) in the polynomial.

This means that the graph of the function will touch the x-axis at x = -5, but not cross it, and the behavior of the graph near x = -5 will be determined by the degree of the zero (which is 3 in this case).

To know more about graph refer here:

https://brainly.com/question/17267403

#SPJ11

Suppose C is any curve from (0,0,0) to (1,1,1) and F (x, y, z) = (1z + 5y) i + (1z + 5x)j + (1y + 1x)k. After confirming that F is conservative, compute a potential function f for F with constant term 0.

Answers

The potential function f(x, y, z) evaluated at the endpoints of C gives the same result:

f(1, 1, 1) - f(0, 0, 0) = (1 + 5 + 5 + 1/2 + 1/2) - 0 = 12

This confirms that f is indeed the potential function for F.

How to confirm that F is conservative?

To confirm that F is conservative, we need to check if its curl is zero. The curl of F is given by:

[tex]curl(F) = (∂F_z/∂y - ∂F_y/∂z) i + (∂F_x/∂z - ∂F_z/∂x) j + (∂F_y/∂x - ∂F_x/∂y) k[/tex]

Substituting F(x, y, z) = (1z + 5y) i + (1z + 5x)j + (1y + 1x)k into the above equation, we get:

curl(F) = 0i + 0j + 0k

The potential function f for F, we need to integrate F along any path from (0,0,0) to (1,1,1). Let C be the path given by the line segment connecting (0,0,0) and (1,1,1).

The parametric equations of C are:

x = ty = tz = t

where 0 ≤ t ≤ 1.

We need to evaluate the line integral ∫CF.dr, where r(t) = ti + tj + tk is the position vector of C at time t. The potential function f is defined as the line integral of F from (0,0,0) to (x,y,z), so we need to find an antiderivative of F to evaluate this integral.

The antiderivative of F is:

[tex]f(x, y, z) = z + 5xy + 5xz + (1/2)y^2 + (1/2)x^2 + C[/tex]

where C is a constant of integration. We want f to have a constant term of 0, so we choose C = 0.

[tex]f(x, y, z) = z + 5xy + 5xz + (1/2)y^2 + (1/2)x^2[/tex]

Now we can evaluate the line integral ∫CF.dr by substituting the parametric equations of C into F and taking the dot product with the differential of r(t):

[tex]F(r(t)).dr/dt = ((t+5t) i + (t+5t)j + (t+t)k) . (i+j+k) dt = (7t) dt[/tex]

Integrating from t=0 to t=1, we get:

[tex]∫CF.dr = ∫0^1 7t dt = 7/2[/tex]

Learn more about F is conservative

brainly.com/question/24314070

#SPJ11

Solve for x trigonometry

Answers

Step-by-step explanation:

We are given an angle opposite of the side length x and the hypotenuse 10.

Use SOHCAHTOA, use Sin

[tex] \sin( \alpha ) = \frac{o}{h} [/tex]

We the angle is 20

and the hypotenuse is 10 and the opposite is x.

[tex] \sin(20) = \frac{x}{10} [/tex]

[tex]10 \sin(20) = x[/tex]

And we get

[tex]x = 3.42[/tex]

Apply the Distributive Property to the right side.
12
enter your response herex

enter your response here ​(Type integers or​ fractions.)

Answers

The rewritten expression of 12 using the distributive property is 3(2 + 2)

Rewriting the equation using the distributive property.

From the question, we have the following parameters that can be used in our computation:

12 distributive property

This means that

12

Express as 6 + 6

So, we have

12 = 6 + 6

Factor out 3 from the equation

So, we have

12 = 3(2 + 2)

The above equation has been rewritten using the distributive property.

Hence, the rewritten expression using the distributive property is 3(2 + 2)

Read more about distributive property at

brainly.com/question/2807928

#SPJ1

What is the vertex and x-intercepts of -6x^2-50x+3085. 25

Answers

The vertex and x-intercepts of -6x^2-50x+3085. 25 are approximately -42.60 and 30.97.

To find the vertex and x-intercepts of the quadratic function -6x^2-50x+3085.25, we first need to express it in standard form -6x^2-50x+3085.25 = -6(x^2+8.33x-514.21)

So the x-intercepts are approximately -42.60 and 30.97.

We can complete the square to find the vertex of the parabola:

-6(x^2+8.33x-514.21) = -6[(x+4.165)^2-575.641]

-6(x^2+8.33x-514.21) = -6(x+4.165)^2+3453.844

So the vertex is at (-4.165, 575.844).

To find the x-intercepts, we can set y = 0 and solve for x:

-6x^2-50x+3085.25 = 0

Dividing both sides by -2.25 to simplify, we get:

2.6667x^2+22.2222x-1372.2222 = 0

Using the quadratic formula, we get:

x = (-22.2222 ± sqrt(22.2222^2-4(2.6667)(-1372.2222))) / (2(2.6667))

x = (-22.2222 ± sqrt(37511.1116)) / 5.3334

x = (-22.2222 ± 193.7262) / 5.3334

So the x-intercepts are approximately -42.60 and 30.97.

To learn more about “vertex” refer to the https://brainly.com/question/29476657

#SPJ11

Ranjan is driving to Salt Lake City. His car gets 35. 5 miles per gallon of gasoline. Ranjan starts with his tank full. So far he has made two stops. Each time he stops, ranjan adds gas until his car is full again. At the first stop ranjan adds 6. 7 gallons of gas. At he second stop he adds 3. 4 gallons of gas. How many miles has ranjas drivin so far

Answers

After calculating the distance, Ranjan has driven 358.55 miles so far.

To solve this problem, we need to use the formula:

distance = fuel efficiency x fuel consumed

Let's start by calculating the total fuel consumed. At the first stop, Ranjan adds 6.7 gallons of gas, which means he consumed 6.7 gallons of gas since his tank was full at the beginning of the trip. At the second stop, he adds 3.4 gallons of gas, which means he consumed 3.4 gallons of gas between the first and second stops. Therefore, the total fuel consumed is:

6.7 + 3.4 = 10.1 gallons

Now we can calculate the distance driven using the fuel efficiency of 35.5 miles per gallon:

distance = 35.5 miles/gallon x 10.1 gallons = 358.55 miles

Therefore, Ranjan has driven 358.55 miles so far.

To know more about distance, visit:

https://brainly.com/question/15172156#

#SPJ11

The number of enterprise instant messaging (IM) accounts is projected to grow according to the function N(t) = 2.97t2 + 11.32t + 59.2 (0 ≤ t ≤ 5) where N(t) is measured in millions and t in years, with t = 0 corresponding to 2006. (a) How many enterprise IM accounts were there in 2006? million (b) What was the expected number of enterprise IM accounts in 2009? million

Answers

There were 59.2 million enterprise IM accounts in 2006 and the expected number of enterprise IM accounts in 2009 was 119.89 million.

(a) To find the number of enterprise IM accounts in 2006, we need to evaluate

N(t) at t = 0: N(0) = 2.97(0)^2 + 11.32(0) + 59.2

N(0) = 0 + 0 + 59.2

N(0) = 59.2 million

So, there were 59.2 million enterprise IM accounts in 2006.

(b) To find the expected number of enterprise IM accounts in 2009, we need to evaluate

N(t) at t = 3 (since 2009 corresponds to t = 3): N(3) = 2.97(3)^2 + 11.32(3) + 59.2

N(3) = 2.97(9) + 33.96 + 59.2

N(3) = 26.73 + 33.96 + 59.2

N(3) = 119.89 million

So, the expected number of enterprise IM accounts in 2009 was 119.89 million.

Learn more about calculas,

https://brainly.com/question/29200987

#SPJ11

Drag each tile to its equivalent measure, rounded to the nearest tenth.



19. 810. 222. 715. 4



Measure Equivalent



4 in.



cm



7 kg



lb



6 gal



L



65 ft



m

Answers

The given value of 19 is not a unit of measurement, so it cannot be converted to an equivalent measure.

How to drag each tile to its equivalent measure, rounded to the nearest tenth?

4 in. - 10.2 cm

7 kg - 15.4 lb

6 gal - 22.7 L

5 ft - 1.5 m

Note: The given value of 19 is not a unit of measurement, so it cannot be converted to an equivalent measure.

Learn more about measurement

brainly.com/question/4725561

#SPJ11

Find the area of the regular polygon. Round your answer to the nearest whole number of square units.

The area is about square units. ​

Answers

The area of the regular pentagon is about 9 square units.

To find the area of a regular polygon, we need to know the length of the apothem and the perimeter of the polygon. The apothem is the distance from the center of the polygon to the midpoint of one of its sides, and the perimeter is the sum of the lengths of all the sides.

Since the polygon is regular, all of its sides have the same length. Let's call that length "s". We also know that the polygon has 5 sides, so it is a pentagon. To find the perimeter, we can simply multiply the length of one side by the number of sides:

Perimeter = 5s

Now, to find the apothem, we can use the formula:

Apothem = (s/2) x tan(180/n)

Where "n" is the number of sides. For our pentagon, n = 5, so we have:

Apothem = (s/2) x tan(36)

We can simplify this a bit by noting that tan(36) is equal to approximately 0.7265. So we have:

Apothem = (s/2) x 0.7265

Now we have everything we need to find the area. The formula for the area of a regular polygon is:

Area = (1/2) x Perimeter x Apothem

Substituting in the values we found earlier, we have:

Area = (1/2) x 5s x (s/2) x 0.7265

Simplifying this expression, we get:

Area = (s^2 x 1.8176)

Rounding to the nearest whole number of square units, we have:

Area = 9

So the area of the regular pentagon is about 9 square units.

To know more about pentagon, refer here:

https://brainly.com/question/27476#

#SPJ11

Other Questions
2. Which of the following is TRUE about block design?I. The random assignment of units to treatments in a block design is accomplished separately in each block.II. Matched pairs design is not a type of block design.III. The purpose of blocking is to increase variation in results. I only III only I and II only II and III only I, II, and III Do you think humans will ever be able to forecast severe weather with 100% accuracy? What challenges do we face in developing technology that can do so? Besides forecasting tools, what other technology is needed to prevent severe storms from causing disasters? Do you think all countries have the financial capability to support the development and operation of these technologies? If not, do you think there is an ethical obligation for countries with more financial resources to help poorer countries access these technologies? ANSWER CORRECTLY. ANSWER FAST PLSSSS The company pays for its insurance policies 12 months in advance. Its most recent payment was made on November 1, 2021. The cost of this policy was slightly higher than the cost of coverage for the previous 12 months. Prepaid insurance-$6,600Insurance expense-$5,900Prepare the adiusting entry to account for the expiration of the company's insurance policies during December. How do I calculate how much insurance has expired? Let f(x) = Show that there is no value c E (1,4) such that f'(c) = f(4) f(1)/4-1. Why is this not a contradiction of the Mean Value Theorem? On March 1 a commodity's spot price is $60 and its August futures price is $59. On July 1 the spot price is $64 and theAugust futures price is $63. 50. A company entered into futures contracts on March 1 to hedge its purchase of thecommodity on July 1. It closed out its position on July 1. What is the effective price (after taking account of hedging) paidby the company? 3. Study the quotation, and answer the questions. (8 points) "Freedom is never voluntarily given by the oppressor; it must be demanded by the oppressed. Frankly, I have yet to engage in a direct-action campaign that was 'well timed' in the view of those who have not suffered unduly from the disease of segregation. For years now I have heard the word 'Wait!' It rings in the ear of every Negro with piercing familiarity. This 'Wait' has almost always meant 'Never." -Martin Luther King, Letter from a Birmingham Jail What does this text tell us about why Martin Luther King was in jail, and what he hoped to accomplish? In LMN, m = 59 inches, n = 35 inches and L=82. Find N, to the nearest degree A circle is circumscribed around a regular octagon with side lemgths of 10 feet. Another circle is inscribed inside the octagon. Find the area. Of the ring created by the two circles. Round the respective radii of the circles to two decimals before calculating the area let s be a set. suppose that relation r on s is both symmetric and antisymmetric. prove that r rdiagonal The base of a triangular prisms has an area of 18 square inches if the height of the prism is 9. 5 inches then what what is the volume of the prism for each of the following write whether it is homozygous dominant, heterozygous or homozygous recessive.AA____ gg_____Pp____ Ii______tt_____ TT______ 18. Mr. Kamau wishes to buy some items for his son and daughter. The son's item costs sh. 324 whilethe daughter item costs sh. 220 each. Mr. Kamau would like to give each of them equal amount ofmoney.a) How many items will each person buys. ASAP Big test Question 7 (1 point) SavedThe subscript numbers in covalent compounds can be determined by_____Question 7 options:the number of the column in which the element is foundthe addition of the charges on each metalthe prefixes used in the written name of the compoundsubtraction of the charges on the metal (12.7)2. A swimming pool is in the shape of a rectangularprism with a horizontal cross-section 10 feet by 20feet. The pool is 5 feet deep and filled to capacity.Water has a density of approximately 60 poundsper cubic footWhat is the approximate mass of water in the pool?A. 8,000 lb.B.12,500 lb.C16,700 lb.D. 60,000 lb. An investment of $4000 is deposited into an account in which interest is compounded continuously. complete the table by filling in the amounts to which the investment grows at the indicated interest rates. (round your answers to the nearest cent.)t = 4 years Which phrase describes the molarity of a solution?. Mr. Smith invested $2500 in a savings account that earns 3% interest compoundedannually. Find the following:1. Is this exponential growth or exponential decay?2. Domain3. Range4. Y-intercept5. Function Rule What is an employee incentive aimed at young parents?Question 23 options:employee college reimbursementcompany provided child careemployee gym membershipdiscounts at expensive restaurants PLEASE HELP! For a group of objects made of the same material, the weight of an object varies directly with its volume. If an object that has a volume of 20 cubic inches weighs 28 ounces, what is the constant of variation?205/77/528 Gazza and Julia have each cut a rectangle out of paper. One side is 10 cm. The other side is n cm. (a) They write down expressions for the perimeter of the rectangle. Julia writes Gazza writes 2n+20 2(n + 10) Put a circle around the correct statement below. Julia is correct and Gazza is wrong.Gazza is correct and julia is wrong.Both are correct.Both are wrong.